Difference between revisions of "2021 AMC 10A Problems/Problem 2"

m (Solution 4.2 (Plug in the Answer Choices))
m (Video Solution 4)
 
(12 intermediate revisions by 3 users not shown)
Line 4: Line 4:
 
<math>\textbf{(A)} ~600 \qquad\textbf{(B)} ~650 \qquad\textbf{(C)} ~1950 \qquad\textbf{(D)} ~2000\qquad\textbf{(E)} ~2050</math>
 
<math>\textbf{(A)} ~600 \qquad\textbf{(B)} ~650 \qquad\textbf{(C)} ~1950 \qquad\textbf{(D)} ~2000\qquad\textbf{(E)} ~2050</math>
  
==Solution 1==
+
==Solution 1 (Two Variables)==
The following system of equations can be formed with <math>p</math> representing the number of students in Portia's high school and <math>l</math> representing the number of students in Lara's high school.
+
The following system of equations can be formed with <math>P</math> representing the number of students in Portia's high school and <math>L</math> representing the number of students in Lara's high school:
<cmath>p=3l</cmath>
+
<cmath>\begin{align*}
<cmath>p+l=2600</cmath>
+
P&=3L, \\
Substituting <math>p</math> with <math>3l</math> we get <math>4l=2600</math>. Solving for <math>l</math>, we get <math>l=650</math>. Since we need to find <math>p</math> we multiply <math>650</math> by 3 to get <math>p=1950</math>, which is <math>\boxed{\text{C}}</math>
+
P+L&=2600.
 +
\end{align*}</cmath>
 +
Substituting <math>P=3L</math> gives <math>4L=2600.</math> Solving for <math>L</math> gives <math>L=650.</math> Since we need to find <math>P,</math> we multiply <math>650</math> by <math>3</math> to get <math>P=\boxed{\textbf{(C)} ~1950}.</math>
  
-happykeeper
+
~happykeeper (Solution)
 +
 
 +
~MRENTHUSIASM (Reformatting)
  
 
==Solution 2 (One Variable)==
 
==Solution 2 (One Variable)==
Line 18: Line 22:
  
 
==Solution 3 (Arithmetic)==
 
==Solution 3 (Arithmetic)==
Clearly, <math>2600</math> students is <math>4</math> times as many students as Lara's high school. Therefore, Lara's high school has <math>2600\div4=650</math> students, and Portia's high school has <math>650\cdot3=\boxed{\textbf{(C)} ~1950}</math> students.
+
Clearly, <math>2600</math> is <math>4</math> times the number of students in Lara's high school. Therefore, Lara's high school has <math>2600\div4=650</math> students, and Portia's high school has <math>650\cdot3=\boxed{\textbf{(C)} ~1950}</math> students.
  
 
~MRENTHUSIASM
 
~MRENTHUSIASM
  
==Solution 4 (Answer Choices)==
+
==Solution 4 (Observations)==
===Solution 4.1 (Quick Inspection)===
 
 
The number of students in Portia's high school must be a multiple of <math>3.</math> This eliminates <math>\textbf{(B)},\textbf{(D)},</math> and <math>\textbf{(E)}.</math> Since <math>\textbf{(A)}</math> is too small (as it is clear that <math>600+\frac{600}{3}<2600</math>), we are left with <math>\boxed{\textbf{(C)} ~1950}.</math>
 
The number of students in Portia's high school must be a multiple of <math>3.</math> This eliminates <math>\textbf{(B)},\textbf{(D)},</math> and <math>\textbf{(E)}.</math> Since <math>\textbf{(A)}</math> is too small (as it is clear that <math>600+\frac{600}{3}<2600</math>), we are left with <math>\boxed{\textbf{(C)} ~1950}.</math>
  
 
~MRENTHUSIASM
 
~MRENTHUSIASM
  
===Solution 4.2 (Plug in the Answer Choices)===
+
==Video Solutions==
For <math>\textbf{(A)},</math> we have <math>600+\frac{600}{3}=800\neq2600.</math> So, <math>\textbf{(A)}</math> is incorrect.
+
===Video Solution 1 (Very Fast & Simple)===
 
 
For <math>\textbf{(B)},</math> we have <math>650+\frac{650}{3}=866\frac{2}{3}\neq2600.</math> So, <math>\textbf{(B)}</math> is incorrect.
 
 
 
For <math>\textbf{(C)},</math> we have <math>1950+\frac{1950}{3}=2600.</math> So, <math>\boxed{\textbf{(C)} ~1950}</math> is correct.
 
 
 
For completeness, we will check answer choices <math>\textbf{(D)}</math> and <math>\textbf{(E)}.</math>
 
 
 
For <math>\textbf{(D)},</math> we have <math>2000+\frac{2000}{3}=2666\frac{2}{3}\neq2600.</math> So, <math>\textbf{(D)}</math> is incorrect.
 
 
 
For <math>\textbf{(E)},</math> we have <math>2050+\frac{2050}{3}=2733\frac{1}{3}\neq2600.</math> So, <math>\textbf{(E)}</math> is incorrect.
 
 
 
~MRENTHUSIASM
 
 
 
==Video Solution (Very fast & Simple)==
 
 
https://youtu.be/DOtysU-a1B4
 
https://youtu.be/DOtysU-a1B4
  
 
~ Education, the Study of Everything
 
~ Education, the Study of Everything
  
 
+
===Video Solution 2 (Setting Variables)===
==Video Solution #1(Setting Variables) ==
 
  
 
https://youtu.be/qNf6SiIpIsk?t=119
 
https://youtu.be/qNf6SiIpIsk?t=119
 
~ThePuzzlr
 
~ThePuzzlr
  
==Video Solution #2(Solving by equation) ==
+
===Video Solution 3 (Solving by Equation)===
  
 
https://www.youtube.com/watch?v=aOpgeMfvUpE&list=PLexHyfQ8DMuKqltG3cHT7Di4jhVl6L4YJ&index=1
 
https://www.youtube.com/watch?v=aOpgeMfvUpE&list=PLexHyfQ8DMuKqltG3cHT7Di4jhVl6L4YJ&index=1
 
~North America Math Contest Go Go Go
 
~North America Math Contest Go Go Go
  
==Video Solution==
+
===Video Solution 4 by OmegaLearn===
 
https://youtu.be/xXx0iP1tn8k
 
https://youtu.be/xXx0iP1tn8k
  
 
- pi_is_3.14
 
- pi_is_3.14
  
==Video Solution 5==
+
===Video Solution 5===
 
https://youtu.be/GwwDQYqptlQ
 
https://youtu.be/GwwDQYqptlQ
  
 
~savannahsolver
 
~savannahsolver
  
==Video Solution 6==
+
===Video Solution 6===
 
https://youtu.be/50CThrk3RcM?t=66
 
https://youtu.be/50CThrk3RcM?t=66
  
 
~IceMatrix
 
~IceMatrix
  
==Video Solution (Problems 1-3)==
+
===Video Solution 7 (Problems 1-3)===
 
https://youtu.be/CupJpUzKPB0
 
https://youtu.be/CupJpUzKPB0
  
 
~MathWithPi
 
~MathWithPi
 +
 +
===Video Solution 8===
 +
https://youtu.be/slVBYmcDMOI
 +
 +
~The Learning Royal
  
 
==See Also==
 
==See Also==
 
{{AMC10 box|year=2021|ab=A|num-b=1|num-a=3}}
 
{{AMC10 box|year=2021|ab=A|num-b=1|num-a=3}}
 
{{MAA Notice}}
 
{{MAA Notice}}

Latest revision as of 21:59, 28 October 2022

Problem

Portia's high school has $3$ times as many students as Lara's high school. The two high schools have a total of $2600$ students. How many students does Portia's high school have?

$\textbf{(A)} ~600 \qquad\textbf{(B)} ~650 \qquad\textbf{(C)} ~1950 \qquad\textbf{(D)} ~2000\qquad\textbf{(E)} ~2050$

Solution 1 (Two Variables)

The following system of equations can be formed with $P$ representing the number of students in Portia's high school and $L$ representing the number of students in Lara's high school: \begin{align*} P&=3L, \\ P+L&=2600. \end{align*} Substituting $P=3L$ gives $4L=2600.$ Solving for $L$ gives $L=650.$ Since we need to find $P,$ we multiply $650$ by $3$ to get $P=\boxed{\textbf{(C)} ~1950}.$

~happykeeper (Solution)

~MRENTHUSIASM (Reformatting)

Solution 2 (One Variable)

Suppose Lara's high school has $x$ students, so Portia's high school has $3x$ students. We have $x+3x=2600,$ or $4x=2600.$ The answer is \[3x=2600\cdot\frac 34=650\cdot3=\boxed{\textbf{(C)} ~1950}.\]

~MRENTHUSIASM

Solution 3 (Arithmetic)

Clearly, $2600$ is $4$ times the number of students in Lara's high school. Therefore, Lara's high school has $2600\div4=650$ students, and Portia's high school has $650\cdot3=\boxed{\textbf{(C)} ~1950}$ students.

~MRENTHUSIASM

Solution 4 (Observations)

The number of students in Portia's high school must be a multiple of $3.$ This eliminates $\textbf{(B)},\textbf{(D)},$ and $\textbf{(E)}.$ Since $\textbf{(A)}$ is too small (as it is clear that $600+\frac{600}{3}<2600$), we are left with $\boxed{\textbf{(C)} ~1950}.$

~MRENTHUSIASM

Video Solutions

Video Solution 1 (Very Fast & Simple)

https://youtu.be/DOtysU-a1B4

~ Education, the Study of Everything

Video Solution 2 (Setting Variables)

https://youtu.be/qNf6SiIpIsk?t=119 ~ThePuzzlr

Video Solution 3 (Solving by Equation)

https://www.youtube.com/watch?v=aOpgeMfvUpE&list=PLexHyfQ8DMuKqltG3cHT7Di4jhVl6L4YJ&index=1 ~North America Math Contest Go Go Go

Video Solution 4 by OmegaLearn

https://youtu.be/xXx0iP1tn8k

- pi_is_3.14

Video Solution 5

https://youtu.be/GwwDQYqptlQ

~savannahsolver

Video Solution 6

https://youtu.be/50CThrk3RcM?t=66

~IceMatrix

Video Solution 7 (Problems 1-3)

https://youtu.be/CupJpUzKPB0

~MathWithPi

Video Solution 8

https://youtu.be/slVBYmcDMOI

~The Learning Royal

See Also

2021 AMC 10A (ProblemsAnswer KeyResources)
Preceded by
Problem 1
Followed by
Problem 3
1 2 3 4 5 6 7 8 9 10 11 12 13 14 15 16 17 18 19 20 21 22 23 24 25
All AMC 10 Problems and Solutions

The problems on this page are copyrighted by the Mathematical Association of America's American Mathematics Competitions. AMC logo.png